Übung am Fr, 15. Dez. 2006

Antworten
Benutzeravatar
pat
Beiträge: 418
Registriert: 07.10.2006, 16:11
Kontaktdaten:

Übung am Fr, 15. Dez. 2006

Beitrag von pat »

Zuletzt geändert von pat am 14.12.2006, 21:28, insgesamt 1-mal geändert.
Ich hab das Forum lieb, weil es schon so lange da ist und man auch Infos von höheren Semestern bekommt :)

Benutzeravatar
ibi
Dr. h.c.
Beiträge: 443
Registriert: 12.10.2006, 20:34
Wohnort: Kagran / Donaustadt

Re: Übung am Fr, 15. Dez. 2006

Beitrag von ibi »

gracvaloth hat geschrieben:Angabe für Freitag, den 15. Dez. 2006
404

Benutzeravatar
pat
Beiträge: 418
Registriert: 07.10.2006, 16:11
Kontaktdaten:

Beitrag von pat »

Done.

Aber wenn man mal mit meiner Notation vertraut ist, sollte es ja kein Problem sein, Dateien bei einem falsch gesetzen Link aufzufinden...
Ich hab das Forum lieb, weil es schon so lange da ist und man auch Infos von höheren Semestern bekommt :)

Benutzeravatar
ibi
Dr. h.c.
Beiträge: 443
Registriert: 12.10.2006, 20:34
Wohnort: Kagran / Donaustadt

Beitrag von ibi »

Nach 4 Stunden Dat kann man nicht mehr logisch denken. ;)

Benutzeravatar
ManuelO
Beiträge: 118
Registriert: 09.10.2006, 16:07

Beitrag von ManuelO »

hab mich grad bisserl mit der theorie vertraut gemacht;)

thx für die beispiele....

Benutzeravatar
ManuelO
Beiträge: 118
Registriert: 09.10.2006, 16:07

Beitrag von ManuelO »

PS: deine lösung beim beispiel 34 ist falsch...

kontrollier noch mal die randbedingungen für das Y...

Wenn du e einsetzt dann kommt für C_1 nicht 0 herauß!
somit fällt das zeta nicht weg und du bekommst ein längeres ergebnis:
w=(e^{-3} - 1) \cdot \ln(z) \cdot z^3 oda so in der art....

mfg

Benutzeravatar
pat
Beiträge: 418
Registriert: 07.10.2006, 16:11
Kontaktdaten:

Beitrag von pat »

Wieso? ln(e) = 1, demnach steht dort c1 + 1 = 1, also c1 = 0?
Ich hab das Forum lieb, weil es schon so lange da ist und man auch Infos von höheren Semestern bekommt :)

Benutzeravatar
ManuelO
Beiträge: 118
Registriert: 09.10.2006, 16:07

Beitrag von ManuelO »

nein, weil
Y(\ln e)\ne 1
, sondern aus der formel zu berechnen:
Y=\frac{W(z)}{H(z)}
und für z=e einsetzen...

dann bekommst du für C_1=e^{-3}-1 raus...

dann alles wie gewohnt...

wird halt bisserl komplizierter

Gal Martin
Beiträge: 194
Registriert: 11.10.2006, 14:53

Beitrag von Gal Martin »

beim 39er hast du noch einen fehler
die ableitung von f(-3) ist nicht 24 sondern -24
dadurch wird der eine term im ergebnis dann auch negativ aber is nur a vorzeichen :) sonst passt supa danke fürs online stellen wiedermal
WO IS DA GAL?!?! :twisted:
© Wiesinger

Benutzeravatar
ManuelO
Beiträge: 118
Registriert: 09.10.2006, 16:07

Beitrag von ManuelO »

hm unten steht der Betrag in der formel...

sollte also passen wenn du das gemeint hast..

Gal Martin
Beiträge: 194
Registriert: 11.10.2006, 14:53

Beitrag von Gal Martin »

jo grad gesehen das unten hat eh passt nur das minus oben hätte hingehört
WO IS DA GAL?!?! :twisted:
© Wiesinger

Antworten

Zurück zu „Mathematische Methoden der Physik“